Use non-breaking space (~) between 'Figure' and the figure number.
[course.git] / latex / problems / Serway_and_Jewett_8 / problem30.61.tex
1 \begin{problem*}{30.61}
2 Two long, straight wires cross each other perpendicularly as shown in
3 Figure~P30.61.  The wires do not touch.  Find \Part{a} the magnitude
4 and \Part{b} the direction of the magnetic field at point $P$, which
5 is in the same plane as the two wires.  \Part{c} Find the magnetic
6 field at a point $30.0\U{cm}$ above the point of intersection of the
7 wires along the $z$ axis; that is, $30.0\U{cm}$ out of the page,
8 toward you.
9 \begin{center}
10 \begin{asy}
11 import Mechanics;
12 import ElectroMag;
13
14 real d = 6cm;
15
16 real low = 0.3d;
17 real high = 0.5d;
18
19 draw_ijhat(0.8*low*(-1,-1));
20
21 Vector Ix = Current((-low, 0), (high+low)+12pt, dir=0,
22     L=Label("$5.00\U{A}$", position=EndPoint, align=S));
23 Ix.draw();
24 Vector Iy = Current((0, -low), (high+low), dir=90,
25     L=Label("$3.00\U{A}$", position=EndPoint, align=W));
26 Iy.draw();
27
28 pair P = (0.4d, 0.3d);
29 Distance Dh = Distance((0, P.y), P, "$30.0\U{cm}$");
30 Dh.draw();
31 Distance Dv = Distance((P.x, 0), P, "$40.0\U{cm}$");
32 Dv.draw();
33
34 dot("$P$", P, NE);
35 \end{asy}
36 \end{center}
37 \end{problem*}
38
39 \begin{solution}
40 \Part{a}
41 We can use the formula for the magnetic field generated by a long,
42 straight wire to calculate the contribution from each wire.  Letting
43 \emph{out of the page} be positive,
44 \begin{equation}
45   B_P =   \frac{\mu_0 (5.00\U{A})}{2\pi(0.400\U{m})}
46         - \frac{\mu_0 (3.00\U{A})}{2\pi(0.300\U{m})}
47     = \ans{500\U{nT}} \;.  \label{eq:30.61.a}
48 \end{equation}
49
50 \Part{b}
51 Because we picked \emph{out of the page} as the positive direction,
52 the $B_P>0$ in \cref{eq:30.61.a} means the magnetic field points
53 \ans{out of the page}.
54
55 \Part{c}
56 We take the same approach for point $Q$, $30.0\U{cm}$ above the
57 intersection, but using the right hand rule to determine the direction
58 of the magnetic field contributions, we can see that we'll need to use
59 vector addition to determine the resulting magnetic field.
60 \begin{center}
61 \begin{asy}
62 import Mechanics;
63 import ElectroMag;
64
65 real d = 0.5cm;
66
67 Vector Bx = BField(mag=5d, dir=-90,
68     L=Label("$\vect{B}_{xP}$", position=EndPoint));
69 Vector By = BField(mag=3d, dir=0,
70     L=Label("$\vect{B}_{yP}$", position=EndPoint));
71 pair b = Bx.mag*dir(Bx.dir) + By.mag*dir(By.dir);
72 Vector B = BField(mag=length(b), dir=degrees(b),
73     L=Label("$\vect{B}_P$", position=EndPoint));
74 Angle t = Angle(dir(0), (0,0), b, "$\theta_P$");
75 t.draw();
76 Bx.draw();
77 By.draw();
78 B.draw();
79 dot("$P$", (0,0), W);
80
81 draw_ijhat((-3d, -2.5d));
82 \end{asy}
83 \end{center}
84 The contribution from the $x$-aligned wire will be along $-\jhat$,
85 while the contribution from the $y$-aligned wire will be along
86 $\ihat$.  The total magnetic field is
87 \begin{align}
88   \vect{B}_Q
89     &=   \frac{\mu_0 I_x}{2\pi r} \cdot (-\jhat)
90        + \frac{\mu_0 I_y}{2\pi r}\ihat
91     = \frac{\mu_0}{2\pi r}\cdot(I_y\ihat-I_x\jhat)
92     = (2.00\ihat - 3.33\jhat)\U{$\mu$T} \\
93   |\vect{B}_Q| &= \sqrt{2.00^2 + (-3.33)^2}\U{$\mu$T}
94     = \ans{3.89\U{$\mu$T}} \\
95   \theta_Q &= \arctan\p({\frac{-3.33}{2.00}})
96     = \ans{-59.0\dg} \;.
97 \end{align}
98 \end{solution}